LSAT and Law School Admissions Forum

Get expert LSAT preparation and law school admissions advice from PowerScore Test Preparation.

 Administrator
PowerScore Staff
  • PowerScore Staff
  • Posts: 8917
  • Joined: Feb 02, 2011
|
#36511
Complete Question Explanation

Strengthen. The correct answer choice is (B)


The philosopher in this stimulus notes that some of the most ardent philosophical opponents
to democracy have correctly noted that both the inherently best and inherently worst forms of
government are those which concentrate power in the hands of few. Based on this premise, the
philosopher concludes that democracy is a better form of government than rule by few, even though
democracy is consistently mediocre—that is, according to the philosopher, reliable mediocrity is
preferable to the prospect of a worse form of government.

The question stem asks which of the answer choices most helps to justify the philosopher’s
argument—that is, which answer choice lends the greatest strength to the argument that, as a
mediocre but consistent form of government, democracy is preferable to both the best and worse
forms of government.

Answer choice (A): The philosopher’s conclusion is that democracy is the better form of
government, regardless of the preference of the majority.

Answer choice (B): This is the correct answer choice. This choice asserts that it is better to
eliminate the worst form of government, even if the cost is not being able to attain the best form of
government.

Answer choice (C): This answer choice is incorrect, because the philosopher’s conclusion excludes
both the best and worst forms of government, in favor of the consistently mediocre form of
government which is democracy.

Answer choice (D): The philosopher’s conclusion is not about which forms of government are the
most equitable, just which ones are “better” than others. Since this answer choice is irrelevant to the
argument in the stimulus, it is incorrect.

Answer choice (E): While the philosopher might agree with the notion that sound philosophical
reasoning provides a better basis than popular preference for decisions about government, this
assertion does not lend any strength to the stimulus’ argument. The philosopher’s theory is that it is
better when choosing a form of government to avoid the worst in favor of safe mediocrity.
 eober
  • Posts: 107
  • Joined: Jul 24, 2014
|
#16398
Hi,

Could you provide an explanation on why we chose answer choice B? I don't understand how it helps the argument "democracy is a mediocre form of government and it is a better choice than rule by the few".I understand that avoiding inherently worst is preferred but why does the answer choice states that avoiding the inherently worst is better than seeking to "attain the best"?

Thanks!
 David Boyle
PowerScore Staff
  • PowerScore Staff
  • Posts: 836
  • Joined: Jun 07, 2013
|
#16425
eober wrote:Hi,

Could you provide an explanation on why we chose answer choice B? I don't understand how it helps the argument "democracy is a mediocre form of government and it is a better choice than rule by the few".I understand that avoiding inherently worst is preferred but why does the answer choice states that avoiding the inherently worst is better than seeking to "attain the best"?

Thanks!
Hello,

It is because rule by few includes "the inherently best", from the stimulus. So, democracy is better only if you are risk-averse and want to avoid the "worst" that rule by few has, and if you don't care that you lose the "best" that rule by few has.

David
 eober
  • Posts: 107
  • Joined: Jul 24, 2014
|
#16439
Hi,

But both inherently best and inherently worst are related to the givernment that concentrate political power in the hands of few. So both are to be avoided, but answer choice says that one is better than the other?
 David Boyle
PowerScore Staff
  • PowerScore Staff
  • Posts: 836
  • Joined: Jun 07, 2013
|
#16594
eober wrote:Hi,

But both inherently best and inherently worst are related to the givernment that concentrate political power in the hands of few. So both are to be avoided, but answer choice says that one is better than the other?
Hello eober,

Yes, answer choice B states a preference for risk avoidance ("avoiding the worst") rather than seeking the best. (And no, the best is not "to be avoided"--who'd want to avoid the best?--, but the worst is certainly to be avoided! even if one has to give up the best, as a result.)

Hope that helps,
David

Get the most out of your LSAT Prep Plus subscription.

Analyze and track your performance with our Testing and Analytics Package.